Questão:
Como é uma transformação galileana das equações de Maxwell?
Keshav Srinivasan
2018-01-09 19:46:33 UTC
view on stackexchange narkive permalink

Na década de 1860, Maxwell formulou o que agora é chamado de equação de Maxwell e descobriu que elas levam a uma conclusão notável: a existência de ondas eletromagnéticas que se propagam a uma velocidade $ c $, que acaba sendo a velocidade da luz, implicando que a luz é uma onda eletromagnética. Agora, o fato de que as equações de Maxwell predizem a velocidade da luz é $ c $ sugerido a Maxwell e outros que as equações de Maxwell não são realmente verdadeiras em todos os referenciais. Em vez disso, eles pensaram, as equações de Maxwell são exatamente verdadeiras em um quadro, o quadro restante do éter, e em todos os outros quadros eles teriam que ser substituídos por outras equações, equações que eram invariantes sob as transformações de Galileu, a fim de se conformar ao princípio da relatividade. Essas outras equações implicavam que a velocidade da luz em outros quadros era na verdade $ c + v $ ou $ c-v $, onde $ v $ é a velocidade do éter. Mas então o experimento de Michelson-Morley, que pretendia encontrar a velocidade $ v $ do éter, acabou mostrando que a velocidade da luz era $ c $ em todos os quadros, aparentemente contradizendo o princípio da relatividade. Mas Einstein mostrou que isso não contradiz o princípio da relatividade, é apenas que você precisa repensar suas noções de espaço e tempo.

Mas minha pergunta é: quais são as equações que as pessoas pensavam serem verdadeiras em quadros diferentes do éter? Colocando de outra forma, quais são as equações que você obtém ao aplicar uma transformação de Galileu às equações de Maxwell? (Ao contrário de uma transformação de Lorentz que deixa as equações de Maxwell inalteradas.)

Na verdade, já vi as equações obtidas antes. Eles foram formulados por algum físico do século 19, talvez Hertz ou Heaviside, e envolvem a adição de termos dependentes da velocidade à lei de Ampère-Maxwell e à lei de Faraday. (Isso depende da velocidade do éter.) Mas não me lembro dos detalhes.

Veja https://physics.stackexchange.com/q/79710/
As equações de Maxwell têm dois limites galileanos diferentes, um limite elétrico e um limite magnético.Ver Marc De Montigny, Germain Rousseaux, "On the electrodynamics of moving corpos at low velocities," http://arxiv.org/abs/physics/0512200.Também http://physics.stackexchange.com/q/30999/, http://arxiv.org/abs/1112.1466, http://arxiv.org/abs/physics/0606228
Um responda:
John Donne
2018-01-09 21:37:32 UTC
view on stackexchange narkive permalink

Não sou especialista no desenvolvimento histórico do assunto, mas vou oferecer uma derivação.

Considere dois referenciais $ S $ e $ S '$, e suponha que $ S' $ se mova com velocidade $ \ textbf v $ em relação a $ S $. As coordenadas em $ S $ e $ S '$ são relacionadas por uma transformação da Galiléia: $$ \ begin {cases} t '= t \\ \ textbf x' = \ textbf x- \ textbf vt \ end {cases} $$ Para descobrir como os campos se transformam, notamos que uma transformação de Lorentz se reduz a uma transformação de Galiléia no limite $ c \ to \ infty $. Na verdade, sob uma transformação de Lorentz os campos se transformam como: $$ \ begin {cases} \ textbf E '= \ gamma (\ textbf E + \ textbf v \ times \ textbf B) - (\ gamma-1) (\ textbf E \ cdot \ hat {\ textbf {v}}) \ hat {\ textbf { v}} \\ \ textbf B '= \ gamma \ left (\ textbf B - \ frac {1} {c ^ 2} \ textbf v \ times \ textbf E \ right) - (\ gamma-1) (\ textbf B \ cdot \ hat {\ textbf {v}}) \ hat {\ textbf {v}} \\ \ end {casos} $$ Tomando o limite $ c \ to \ infty $ de forma que $ \ gamma \ to 1 $, obtemos as transformações galilianas dos campos: $$ \ begin {cases} \ textbf E '= \ textbf E + \ textbf v \ times \ textbf B \\ \ textbf B '= \ textbf B \\ \ end {casos} $$ Podemos então inverter a transformação enviando $ \ textbf v \ para - \ textbf v $: $$ \ begin {cases} \ textbf E = \ textbf E '- \ textbf v \ times \ textbf B' \\ \ textbf B = \ textbf B '\\ \ end {casos} $$ Pelo mesmo raciocínio, pode-se obter a transformação galiliana das fontes: $$ \ begin {cases} \ textbf J = \ textbf J '+ \ rho' \ textbf v \\ \ rho = \ rho '\\ \ end {casos} $$ Sabemos que os campos e fontes satisfazem as equações de Maxwell em $ S $: $$ \ begin {cases} \ nabla \ cdot \ textbf E = \ rho / \ epsilon_0 \\ \ nabla \ cdot \ textbf B = 0 \\ \ nabla \ times \ textbf E = - \ frac {\ partial \ textbf B} {\ partial t} \\ \ nabla \ times \ textbf B = \ mu_0 \ left (\ textbf J + \ epsilon_0 \ frac {\ partial \ textbf E} {\ partial t} \ right) \\ \ end {casos} $$ Substituindo os campos e fontes em $ S $ por aqueles em $ S '$ obtemos: $$ \ begin {cases} \ nabla \ cdot \ textbf (\ textbf E '- \ textbf v \ times \ textbf B') = \ rho '/ \ epsilon_0 \\ \ nabla \ cdot \ textbf B '= 0 \\ \ nabla \ times \ textbf (\ textbf E '- \ textbf v \ times \ textbf B') = - \ frac {\ parcial \ textbf B '} {\ parcial t} \\ \ nabla \ times \ textbf B '= \ mu_0 \ left (\ textbf J' + \ rho '\ textbf v + \ epsilon_0 \ frac {\ parcial (\ textbf E' - \ textbf v \ times \ textbf B ')} {\ parcial t} \ direita) \\ \ end {casos} $$ Como última etapa, precisamos substituir os derivativos em $ S $ por derivados em $ S '$. Nós temos: $$ \ begin {cases} \ nabla = \ nabla '\\ \ frac {\ partial} {\ partial t} = \ frac {\ partial} {\ partial t'} - \ textbf v \ cdot \ nabla \ end { casos} $$ Substituindo e removendo os primos e usando cálculo vetorial, obtemos: $$ \ begin {cases} \ nabla \ cdot \ textbf E + \ textbf v \ cdot (\ nabla \ times \ textbf B) = \ rho / \ epsilon_0 \\ \ nabla \ cdot \ textbf B = 0 \\ \ nabla \ times \ textbf E = - \ frac {\ partial \ textbf B} {\ partial t} \\ \ nabla \ times \ textbf B = \ mu_0 \ left (\ textbf J + \ rho \ textbf v + \ epsilon_0 \ frac {\ partial} {\ partial t} (\ textbf E - \ textbf v \ times \ textbf B) - \ epsilon_0 \ textbf v \ cdot \ nabla (\ textbf E - \ textbf v \ times \ textbf B) \ right) \\ \ end {cases} $$

No vácuo, podemos pegar a curva da quarta equação para obter: $$ c ^ 2 \ nabla ^ 2 \ textbf B = \ frac {\ parcial ^ 2 \ textbf B} {\ parcial t ^ 2} + (\ textbf v \ cdot \ nabla) ^ 2 \ textbf B - 2 \ textbf v \ cdot \ nabla \ left (\ frac {\ partial \ textbf B} {\ partial t} \ right) $$ Substituindo uma solução de onda da forma $ \ textbf B \ sim \ exp {i (\ textbf k \ cdot \ textbf x - \ omega t)} $ Obtemos uma equação para $ \ omega $, que podemos resolver para obter: $$ \ omega = - \ textbf v \ cdot \ textbf k \ pm c | \ textbf k | $$ Portanto, a velocidade de propagação é a velocidade do grupo: $$ \ frac {\ partial \ omega} {\ partial \ textbf k} = - \ textbf v \ pm c \ hat {\ textbf {k}} $$ que fornece o $ c \ pm v $ esperado com uma escolha apropriada de $ \ textbf v $ e $ \ textbf k $.

uau, muito impressionante, +1.-NN


Estas perguntas e respostas foram traduzidas automaticamente do idioma inglês.O conteúdo original está disponível em stackexchange, que agradecemos pela licença cc by-sa 3.0 sob a qual é distribuído.
Loading...